Monday, December 23, 2013

Causal Reasoning Problem Set Answer Key


All answer keys in this book will indicate the source of the question by giving the month and year the
LSAT was originally administered, the Logical Reasoning section number, and the question number within
that section. Each LSAT has two Logical Reasoning sections, and so the Section 1 and Section 2
designators will refer to the first or second Logical Reasoning section in the test, not the physical section
number of the booklet.
Question #1. Weaken-CE. December 2000 LSAT, Section 1, #25. The correct answer choice is (D)
The stimulus commits the classic error of assuming that because two events occur simultaneously that one
must cause the other. The phrase used to indicate the causality is “responsible for.”
D = anticollision device
SD = sudden disappearance of key information
C                           E
D----------------------> SD
The question stem asks you to weaken the argument, and according to the “How to Attack a Causal
Conclusion” section you should be on the lookout for one of several primary methods of attacking the
argument.
Answer choice (A): This answer presents another effect of the cause, but this additional effect does not
weaken the argument. To analogize this answer to the argument, imagine a scenario where a speaker
concludes that playing football makes a person more prone to sustaining a leg injury. Would suggesting
that playing football makes a person more prone to a head injury (another effect) undermine the first
statement? No.
Answer choice (B): This is an Opposite answer that supports the conclusion. By showing that the key
information did not disappear prior to the appearance of the anticollision device, the argument is
strengthened because the likelihood that the device is at fault is increased.
Answer choice (C): This information has no effect on determining if the device causes the information to
disappear from the screen because it references an event that has yet to occur.
Answer choice (D): This is the correct answer, and this answer falls into the third category for weakening
a causal argument: “Show that although the effect exists, the cause did not occur.” In this instance, the
effect of information disappearing from the screen occurred prior to the creation of the supposed causal
agent, the anticollision device.
Answer choice (E): This answer choice has no impact on the argument. We cannot make a judgment
based on the size of the airport because the argument did not mention airport size or anything directly
related to airport size.
Question #2. Must-CE. December 2001 LSAT, Section 1, #4. The correct answer choice is (C)
The causal relationship in this problem appears in the premise, and the argument is structured as follows:
Premise: Most antidepressant drugs cause weight gain.
Premise: Dieting can help reduce the amount of weight gained while taking such antidepressants
Conclusion: Some weight gain is unlikely to be preventable.
Note that the causal premise specifically states that “most” antidepressants cause weight gain, not
necessarily all antidepressants. Also, the second premise specifically refers to antidepressants causing
weight gain (the use of “such” indicates this). The second premise also indicates that the amount gained
can be reduced, not that dieting can stop weight gain. Perhaps the antidepressants cause a twenty pound
weight gain, but dieting can reduce that to a ten pound total gain.
The question stem is a Must Be True, and thus you must accept the stimulus information and find an
answer that is proven by that information.
Answer choice (A): This is an Exaggerated answer. The stimulus indicates that most antidepressants cause
weight gain, leaving open the possibility that some do not. This answer choice references any
antidepressant drug. Further, the stimulus does not address the role of a physician or the advisability of
prescribing certain drugs under certain conditions. The benefits of prescribing an antidepressant that causes
weight gain to an overweight patient may well outweigh the negatives (pun intended).
Answer choice (B): This is also an Exaggerated answer. The stimulus allows for antidepressants that do
not cause weight gain.
Answer choice (C): This is the correct answer. Some individuals taking antidepressants that cause weight
gain will gain weight even though dieting can reduce the amount of the gain.
Answer choice (D): This is an Opposite answer. The stimulus and correct answer both indicate that people
taking the weight gain-causing antidepressants will gain weight regardless of whether they diet. Thus, the
weight gain cannot be attributed to a lack of dieting.
Answer choice (E): This answer is too strong. Not all patients necessarily take antidepressants that cause
weight gain, so those that do not might not need to diet to maintain their weight.
Question #3. Weaken-CE. December 2003 LSAT, Section 2, #6. The correct answer choice is (C)
The premise contains information concerning a rise in the number of calls involving violent crimes
compared to last year. This is where smart LSAT reading comes into play: does the argument say there is
more crime, or does it say there are more calls reporting crime? Recognizing the difference is critical for
successfully solving this problem. The conclusion about citizens being more likely to be victimized by a
violent crime indicates the author believes the following causal relationship:
GNC = greater number of violent crimes
MC = more calls involving violent crimes
C                           E
GNC-------------------> MC
Literally, the author believes that there are more violent crimes and therefore the police are responding to
more violent crime calls.
The question stem asks you to weaken the argument, and the correct answer falls into one of the five basic
methods for weakening a causal argument.
Answer choice (A): This is an Opposite answer that strengthens the argument.
Answer choice (B): Because the argument is about “the average citizen of this town,” information about
victims of a certain age is not relevant.
Answer choice (C): This is the correct answer. By showing that people are more willing to report crimes
(and thus call them in for response), an alternate cause for the rise in the number of calls is given.
Answer choice (D): This answer only addresses an effect of the concern over crime, and does not address
the causal relationship that underlies the argument.
Answer choice (E): This answer does not address a possible rise in crime or the reasons for the rise in
responses to calls involving violent crime.
Question #4. Weaken-CE. October 1999 LSAT, Section 2, #24. The correct answer choice is (B)
The premises contain correlations, and the conclusion makes a causal claim:
PC = adequate prenatal care
DR = decrease risk of low birth weight babies
C                      E
PC----------------> DR
The question stem asks you to weaken the argument, and the correct answer falls into one of the five basic
methods for weakening a causal argument.

Answer choice (A): The conclusion specifically states that mothers who had received adequate prenatal
care were less likely to have low birth weight babies than mothers who had received inadequate prenatal
care. Thus, although mothers who received inadequate prenatal care have a higher likelihood of having
low birth weight babies, this likelihood still allows for many babies to be born of normal weight. In a later
chapter we will explore the ways the LSAT uses numbers and statistics to confuse test takers, but for now,
consider this analogy: The Detroit Tigers are more likely to lose a baseball game than any other team, but
even so, they can still win a number of games. In the same way, the aforementioned mothers may be more
likely to have low birth weight babies, but they can still give birth to babies of normal weight. Hence,
answer choice (A) does not attack the argument.
Answer choice (B): This is the correct answer. The answer choice falls into the category of “Showing a
statistical problem exists with the data used to make the causal statement.” By indicating that all mothers
without prenatal care records are automatically classified as mothers receiving inadequate prenatal care, the
answer undermines the relationship in the argument because the data used to make the conclusion is
unreliable.
Answer choice (C): The conclusion is about low birth weight babies, not premature babies. Even if low
birth weight babies were routinely classified as premature, that would not affect the conclusion.
Answer choice (D): Similar to answer choice (A), the likelihoods discussed in the stimulus allow for this
possibility. Hence, this answer cannot hurt the argument.
Answer choice (E): If anything, this answer strengthens the argument since it shows that adequate prenatal
care has a powerful positive effect.
Question #5. Flaw-CE. October 2003 LSAT, Section 2, #13. The correct answer choice is (C)
This Flaw in the Reasoning problem is very similar to the Flaw problem discussed in the chapter (problem
#1). A correlation involving theta waves, TV watching, and personality disorders is presented in the
premises, and then the author concludes that watching too much TV causes a rise in the risk of developing
a personality disorder to rise.
Answer choice (A): Although “personality disorders” are left largely undefined (which is acceptable), the
term is not used ambiguously.
Answer choice (B): This is not an error because the author is not obligated to define theta brain waves in
order to make the argument understandable.
Answer choice (C): This is the correct answer.
Answer choice (D): There is no information to prove that the sample of data used was unrepresentative.
Although the researcher says, “my data show...” it is possible the researcher’s data are extensive and
representative.
Answer choice (E): This is a Shell Game answer because the researcher infers that the reverse is true.
Question #6. Weaken-CE. December 2000 LSAT, Section 2, #7. The correct answer choice is (A)
The heart of the argument is a causal claim that the reason today’s newspapers are full of stories about
violent crime is that violent crime has now risen to a point where it is out of control.
MVC = more violent crimes
NFS = more news stories about violent crimes
C                        E
MVC-----------------> NFS
On the basis of this relationship, the author adds for good measure that one should not leave one’s home!
From a form standpoint, this problem is very similar to #3 in this problem set. This is one of the keys to the
LSAT—you must recognize the patterns that exist within the test and then capitalize on them when they
appear. All the problems in the set include causality. From this point on, you must recognize causality
when it appears and then properly respond to it. Your ability to recognize these forms will give you an
advantage in both speed and confidence, and ultimately raise your score.
Answer choice (A): This is the correct answer. This answer presents an alternate cause to the scenario
presented above, namely that more comprehensive coverage leads to more news stories, not more violent
Answer choice (B): This Opposite answer strengthens the argument.
Answer choice (C): This answer strengthens the argument, if anything.
Answer choice (D): This is an answer that many people select. The answer is incorrect because it fails to
account for other violent crimes beside murder. Indicating that murder comprised a higher percentage of
violent crimes in the old days than today does not address the total number of crimes being committed.
Since the argument concludes that “violent crime is now out of control,” this answer is incorrect. Consider
the following example:
                               Old Days                Today
Total number of murders                3                  1,000
Total number of violent crimes           4                   50,000
Percentage of violent crimes
that are murders                     75%                   2%
In this example, although murder was a higher proportion of the violent crimes in the old days, today there
are many more violent crimes. This shows that the scenario in the answer choice does not have to
undermine the argument. In the chapter on Numbers and Percentages we will revisit the concept of
proportion versus total numbers and discuss how the test makers use numerical ideas to attack test takers.
Answer choice (E): The role played by magazines in informing the public does not address why there are
so many stories about violent crime or if violent crime is now out of control.

No comments:

Post a Comment